On a coordinate plane, triangle K J L is shown. Line segment G H goes from side J K to J L. Point K is at (0, 0), point G is at (e, f), point J is at (2 e, 2 f), point H is at (e + d, f), and point L is at (2 d, 0).

To prove part of the triangle midsegment theorem using the diagram, which statement must be shown?
The length of JK equals the length of JL.
The length of GH is half the length of KL.
The slope of JK equals the slope of JL.
The slope of GH is half the slope of KL.

Answers

Answer 1

Based on the triangle midpoint theorem, the statement which must be shown is that: B. the length of GH is half the length of KL.

What is triangle midpoint theorem?

Triangle midpoint theorem states that the line segment which joins the midpoints of two (2) sides of a triangle is parallel to the third side, and it's congruent to one-half of the third side.

Based on the triangle midpoint theorem, we can infer and logically deduce that the statement which must be shown is that the length of GH is half the length of KL.

Read more on triangle midpoint theorem here: https://brainly.com/question/16047906

#SPJ1

On A Coordinate Plane, Triangle K J L Is Shown. Line Segment G H Goes From Side J K To J L. Point K Is
Answer 2

Answer:  B. The length of GH is half the length of KL.

Step-by-step explanation:   TRUST ME!  The answer is correct on Edge.  :)


Related Questions

4. Use the triangle congruence postulates to prove congruence.
Part I: Fill in the blanks to name each triangle congruence postulate.
a. SSS:
b. SAS:
c. ASA:
AAS:
In the triangle

Answers

The triangle congruence postulate is illustrated below

How to illustrate the triangle?

It should be noted that two triangles are congruent if they have the same shape and size.

AAS: The angle-angle-side theorem illustrates that two triangles are congruent if the two angles and a side note between the two angles are identical to that of another triangle.

SSS: This illustrates that the three sides of a triangle are congruent to the corresponding three sides of the other triangle.

ASA: This illustrates that the triangles are congruent when the two angles and the side are the same as the corresponding angles and sides of the other triangle.

SAS: When two sides and the included angle of a triangle are congruent to the corresponding sides and the included angle if a second triangle then it follows the side-angle-side theorem.

Learn more about triangles on:

brainly.com/question/1058720

#SPJ1

30 points plus brainliest if answered right!

Answers

Answer:

The second choice.

Step-by-step explanation:

The length of AC must be equal to 5, since it is horizontally five units. We can rule out the first option.

Length BC cannot be the square root of 16, since that would be exactly 4 units. BC is slanted and does not represent a completely vertical line, so it is not a whole number. We can rule out the fourth option.

Length AB cannot be a whole number, being a slanted line that crosses a total of more than four units. Option three can be ruled out.

Therefore, we are left with the second option.

Answer:

AC = 5, BC = √17, AB = √32

Explanation:

Given points: A(-3, -2), B(1, 2), C(2, -2)

Use distance between two points formula:

[tex]\sf Distance \ between \ two \ points = \sqrt{(x_2 - x_1)^2 + (y_2 - y_1)^2}[/tex]

AC length:

[tex]\sf d = \sqrt{(x_2 - x_1)^2 + (y_2 - y_1)^2} = \sqrt{(2-(-3))^2 + (-2-(-2))^2} = \sqrt{25+ 0} = 5[/tex]

BC length:

[tex]\sf d = \sqrt{(x_2 - x_1)^2 + (y_2 - y_1)^2} = \sqrt{(2-1)^2 + (-2-2)^2} = \sqrt{1 + 16} = \sqrt{17}[/tex]

AB length:

[tex]\sf d = \sqrt{(x_2 - x_1)^2 + (y_2 - y_1)^2} = \sqrt{(-3-1)^2 + (-2-2)^2} = \sqrt{16 + 16} = \sqrt{32}[/tex]

Which properties are necessary to claim that the two prisms are congruent? Check all that apply.


Answers

Answer:

1. The volumes are equal.

2. The base areas are equal.

3. The prisms have the same height.

Step-by-step explanation:

I don't see no answered to check all that apply but here is the answers about properties and two prisms

GIVING BRAINLEST!
Linda collected 1/2 of a stamp album. Becky collected 7 1/3 times as many stamps as Linda. How many
stamp albums were collected by Becky?

O 3 1/3
O 2 1/3
O 3 2/3
O 1 1/3
*

Answers

Answer:

3 2/3

Step-by-step explanation:

To do this question, you would need to multiply 7 1/3 with 1/2. The answer is 3 2/3.

Answer:

3 2/3 albums

Step-by-step explanation:

Let's say that the stamp album has 12 stamps (12 is just an arbitrary number used to make calculation easier). If Linda collects half of the album, it means she has 6 stamps. If that is the case, Becky has 6 * 7 1/3 stamps. Since 7 1/3 is equivalent to 22/3, we can determine that Becky has collected 44 stamps (6*22/3). There are 12 stamps in one album, which means there are 44 stamps in 3 2/3 (11/3) albums:

[tex]\frac{12}{1}=\frac{44}{y} \\12y=44\\y=11/3[/tex]

Since the ratio of stamps to albums is constant, the calculation above demonstrates that the number of albums it takes to have 44 stamps is 3 2/3. Please note that the numbers I used are arbitrary. If you substitute 12 for a variable x, you will get the same result.

Find the probability for the experiment of drawing two marbles at random (without replacement) from a bag containing three green, three yellow, and four red marbles.

The marbles are different colors.

Answers

Answer:

[tex]\frac{11}{15} =0.733333333333[/tex]

Step-by-step explanation:

• Here The sample space S is the set of possible outcomes (ordered pairs of marbles) that we can draw at random (without replacement) from the bag.

Then

[tex]\text{cardS} =P^{2}_{10}=10\times 9=90[/tex]

……………………………………………

Drawing two marbles where the marbles are different colors

means

drawing (1green ,1 yellow) or (1green ,1 red) or (1yellow ,1 red)

Remark: the order intervene

=========================

•• Let E be the event “Drawing two marbles where the marbles are different colors”.

CardE = 2×3×3 + 2×3×4 + 2×3×4 = 66  (2 is for the order)

Conclusion:

[tex]p\left( E\right) =\frac{66}{90} =\frac{11}{15} =0.733333333333[/tex]

Method 2 :

[tex]p\left( E\right) =2\times \frac{3}{10} \times \frac{3}{9} +2\times \frac{3}{10} \times \frac{4}{9} +2\times \frac{3}{10} \times \frac{4}{9} =0.733333333333[/tex]

Answer:

[tex]\sf \dfrac{11}{15}[/tex]

Step-by-step explanation:

The bag of marbles contains:

3 green marbles3 yellow marbles4 red marbles

⇒ Total marbles = 3 + 3 + 4 = 10

Probability Formula

[tex]\sf Probability\:of\:an\:event\:occurring = \dfrac{Number\:of\:ways\:it\:can\:occur}{Total\:number\:of\:possible\:outcomes}[/tex]

First draw

[tex]\implies \sf P(green)=\dfrac{3}{10}[/tex]

[tex]\implies \sf P(yellow)=\dfrac{3}{10}[/tex]

[tex]\implies \sf P(red)=\dfrac{4}{10}[/tex]

Second draw

As the first marble is not replaced there are now 9 marbles in the bag.

If the first marble was green, the probability of drawing a yellow is now 3/9 and the probability of drawing a red is now 4/9.

If the first marble was yellow, the probability of drawing a green is now 3/9 and the probability of drawing a red is now 4/9.

If the first marble was red, the probability of drawing a green is now 3/9 and the probability of drawing a yellow is now 3/9.

To find the individual probabilities of picking 2 different colors, multiply the probability of the first draw by the probability of the second draw:

[tex]\implies \sf P(green)\:and\:P(red)=\dfrac{3}{10} \times \dfrac{4}{9}=\dfrac{12}{90}[/tex]

[tex]\implies \sf P(green)\:and\:P(yellow)=\dfrac{3}{10} \times \dfrac{3}{9}=\dfrac{9}{90}[/tex]

[tex]\implies \sf P(yellow)\:and\:P(green)=\dfrac{3}{10} \times \dfrac{3}{9}=\dfrac{9}{90}[/tex]

[tex]\implies \sf P(yellow)\:and\:P(red)=\dfrac{3}{10} \times \dfrac{4}{9}=\dfrac{12}{90}[/tex]

[tex]\implies \sf P(red)\:and\:P(green)=\dfrac{4}{10} \times \dfrac{3}{9}=\dfrac{12}{90}[/tex]

[tex]\implies \sf P(red)\:and\:P(yellow)=\dfrac{4}{10} \times \dfrac{3}{9}=\dfrac{12}{90}[/tex]

To find the probability of drawing two marbles at random and the marbles being different colors, add the individual probabilities listed above:

[tex]\begin{aligned}\implies \sf P(2\:different\:color\:marbles) &=\sf \dfrac{12}{90}+\dfrac{9}{90}+\dfrac{9}{90}+\dfrac{12}{90}+\dfrac{12}{90}+\dfrac{12}{90}\\\\ & = \sf \dfrac{66}{90}\\\\ & =\sf \dfrac{11}{15}\end{aligned}[/tex]

HEEEELPPP MEEEEEEE FRRRRRR!!!!!!! 30POINTS!!!!!

Answers

[tex]\huge\text{Hey there!}[/tex]

[tex]\huge\textbf{Circumference of a circle: }[/tex]

[tex]\bullet\ \large\boxed{\mathsf{2\pi r}}}\\\rightarrow\large\boxed{\textsf{Whereas, } \pi \large\textsf{ is your pi and it is approximately equal to 3.14}}\\\rightarrow\large\boxed{\textsf{ And \underline{r} is the radius of the circle.}}[/tex][tex]\huge\textbf{Area of a triangle:}[/tex]

[tex]\bullet\ \large\boxed{\mathsf{\dfrac{1}{2}bh}}\\\\\rightarrow\large\boxed{\textsf{Whereas, }\mathsf{\dfrac{1}{2}} \large\textsf{ is half your triangle, \underline{b} is your \underline{b}ase, and b is your \underline{h}eight}}[/tex]

[tex]\huge\textbf{Surface area of a rectangular prism: }[/tex]

[tex]\bullet \ \large\boxed{\textsf{2(wl + hl + hw)}}\\\rightarrow\large\boxed{\textsf{Whereas, l is your \underline{l}ength, h is your \underline{h}eight, and w is your \underline{w}idth.}}[/tex]

[tex]\huge\textbf{Volume of a rectangular prism:}[/tex]

[tex]\bullet\ \large\boxed{\textsf{whl = v}}\\\rightarrow\boxed{\large\textsf{Whereas, v is your \underline{v}olume, w is your \underline{w}idth, h is your \underline{h}eight, l is}}\\ \ \large\boxed{\textsf{your \underline{l}ength}}[/tex]

[tex]\huge\text{Good luck on your assignment \& enjoy your day!}[/tex]

~[tex]\frak{Amphitrite1040:)}[/tex]

Let g(x) = f(x+3) for some function f(x). Is g(x) a function? From the previous problem, let the domain of f(x) be [1, 4). What is the domain of g(x)?

Answers

The domain of g(x) is [4, 7)

How to determine the domain of g(x)?

The function is given as:

g(x) = f(x + 3)

Since f(x) is a function, then g(x) is also a function

The domain of f(x) is given as:

[1, 4)

The equivalent of these values in g(x) are

x = 1 + 3 = 4

x = 4 + 3 = 7

Hence, the domain of g(x) is [4, 7)

Read more about domain at

https://brainly.com/question/1770447

#SPJ1

Please help ill give whoever answers the best the brainliest answer :)))

Answers

Answer:

3rd graph from the top.

Step-by-step explanation:

It can't be the top one because the kids shoulder doesn't start at the ground.

It can't be the second or fourth because it shows him not waiting in line.

Therefore it must be the 3rd!

Answer: 3rd option

Step-by-step explanation:

Because the description says that it starts not at 0(from the kid's shoulder), the 1st and 4th options are out.

2nd option is incorrect because the kid had to stand in line so the shoulder height doesn't change(unless he was picking stuff up or something)

That leaves the 3rd option.

The fit line equation for the scatterplot is y = -2.61x+152.51, where x is variable 1 and y is variable 2. according to the equation, what is the expected value of y when x-value is 45?

Answers

The expected value of y when x-value is 45 is 35.06

Any equation which can be put in the form ax + by + c = 0, where a, b and c are real numbers, and a and b are not both zero, is called a linear equation in two variables. For example, y = 2x+3 and 2y = 4x + 9 are two-variable linear equations.

In the given question value of x is given and value of y is asked.

The given equation : y = -2.61x+152.51 Such questions can be solved by substituting the given value of variable in the equation which will help us find the unknown.

Substituting x = 45 in the given equation , we get

y = -2.61 x 45 + 152.51

y =  -117.45 + 152.51

y = 35.06

Thus the expected value of y when x-value is 45 is 35.06

Learn more about Linear equations in two variables here :

https://brainly.com/question/24085666

#SPJ4

In triangle xyz, m∠z > m∠x m∠y. which must be true about △xyz? m∠x m∠z < 90° m∠y > 90° ∠x and∠y are complementary m∠x m∠y < 90°

Answers

m∠X + m∠Y < 90° is true.

How to solve it?

XYZ is a triangle.

We have to find the option that is true about △XYZ.

By angle sum property of a triangle,

The sum of all the interior angles of a triangle is always equal to 180°.

So, ∠X + ∠Y + ∠Z = 180°

Given, m∠Z > m∠X + m∠Y.

This implies that ∠Z is greater than the sum of the angles ∠X and ∠Y.

so, ∠X + ∠Y must be less than 90°.

Hence, we can say that ∠X + ∠Y < 90°

To know more about triangles, visit:

https://brainly.com/question/1968095

#SPJ4

Answer: Option 4 or D. m∠x + m∠y∠90°

Step-by-step explanation: Trust Me! It is correct on Edge!

Q: In triangle xyz, m∠z > m∠x m∠y. which must be true about △xyz?

A: D. m∠x + m∠y∠90°

Kevin earned $80. Out of his earnings, he spent $68/5 on shopping, $51/2 on food, and $22/5 for taxi. Find the remaining amount.

Answers

The amount remaining out the money earned is $37.5

Sum of values and proportion

Given the following parameters

Amount earned by kevin = $80

If he spent $68/5 on shopping, $51/2 on food, and $22/5 for taxi, the total money spent is given as;

Total money spent = 51/2 + 68/5 + 22/5

Total money spent = 51/2 + 90/5

Total money spent = 25.5 + 18

Total money spent = 43.5

Amount remaining = 80. - 43.5

Amount remaining = 37.5

Hence the amount remaining out the money earned is $37.5

Learn more on difference here: https://brainly.com/question/148825

#SPJ1

Checking number 2( Solving inequalities)

Answers

1. x < 20

x - 7 < 13

x < 20

2. x > -7

-3(x + 4) < 9

-3x - 12 < 9

-3x < 21

x > -7

Hope this helps!

Writing Exercises
587. When you convert a number from decimal notation to scientific notation, how do you know if the exponent will be positive or negative?

Answers

Answer:

Therefore, we can say that the exponent will be negative if the number is less than 1 and vice versa.

Step-by-step explanation:

We are asked to explain that when we convert a decimal number to scientific notation how will we know whether the exponent is negative.

If the given number is greater than 1 then the exponent is positive and if less than 1 the exponent is negative.

I need your help :))
Question is:
Make a pattern and explain the rule the pattern follows. Use a minimum of 3 terms within your pattern.

Answers

The pattern {1, 3, 5, ...} is an arithmetic sequence with the following rule:

[tex]a_n = 1 + 2(n - 1)[/tex].

What is an arithmetic sequence?

In an arithmetic sequence, the difference between consecutive terms is always the same, called common difference d.

The nth term of an arithmetic sequence is given by:

[tex]a_n = a_1 + (n - 1)d[/tex]

In which [tex]a_1[/tex] is the first term.

The pattern {1, 3, 5, ...} has first term and common difference given, respectively, by:

[tex]a_1 = 1, d = 2[/tex]

Hence the rule for the pattern is given by:

[tex]a_n = 1 + 2(n - 1)[/tex].

More can be learned about arithmetic sequences at https://brainly.com/question/6561461

#SPJ1

A triangle is shown. One side has a length of 16, another side has a length of 20, and the length of the longest side is unknown.

Janice examines the given triangle and estimates that the longest side has a length of 25 units―if it is a right triangle. How does her estimate compare to the actual length?
It is exactly correct.
It is under by approximately 0.6 units.
It is over by approximately 0.6 units.
It is over by 13 units.

Answers

Janice estimate is under by approximately 0. 6 units. Option B

How to determine the estimate

The longest part is called the hypotenuse

The formula for finding the hypotenuse, we use the Pythagorean theorem

Hypotenuse = square root of both opposite square and adjacent square

We have that,

Hypotenuse = [tex]\sqrt{16^2 + 20^2}[/tex]

Hypotenuse = [tex]\sqrt{256+ 400}[/tex]

Hypotenuse = [tex]\sqrt{656}[/tex]

Find the square root

Hypotenuse = 25. 6

The length of the longest part = 25. 6

Janice estimated it to be 25

Comparing her estimate with the actual length = Actual length - estimate

⇒ 25. 6 - 25

⇒ 0. 6

Therefore, Janice estimate is under by approximately 0. 6 units. Option B

Learn more about Pythagorean theorem here:

https://brainly.com/question/343682

#SPJ1

Answer: B. It is under by approximately 0.6 units.

Step-by-step explanation: Just did it on edge 2023


Find the circumference and the area of a circle with diameter 6 ft.

Answers

Answer:

Circumference = 18.84 ft

Area = 28.26 ft squared

Step-by-step explanation:

C(circumference) = 2 x [tex]\pi[/tex] x r(radius)

radius = 1/2 circumference

1. divide circumference by two to get the radius, which would be three ft.

2. multiply the radius by pie (3.14) and then multiply that by two and you have your circumference, which is 18.84 ft

area = [tex]\pi[/tex] x r(radius)^2

1. find the radius squared, which would be 9 because 3^2 = 3 x 3 which = 9

2. multiply 9 by pie (3.14) and you get the area which =  28.26

Line I is parallel to line M. If the measure of 6 is 75 degrees, what is the measure of 3?
(please look at photo attached)

Answers

In the given diagram, the measure of ∠3 will be 105°.

In the given diagram, ∠3 and ∠6 are consecutive interior angles.

How to form supplementary angles by transversal?

If two parallel lines are cut by a transversal, then the pairs of consecutive interior angles formed are supplementary.

That is,

∠3 + ∠6 = 180°

From the given information,

∠6 = 75°

Then,

∠3 + 75° = 180°

∠3 = 180° - 75°

∠3 = 105°

Hence, the measure of ∠3 will be 105°.

Learn more about the measures of angles here: brainly.com/question/2883630

#SPJ1

How is copying segment similar to copying an angle ?

Answers

Answer:

Copying a line segment or angle is an illustration of transformation. Both are similar, because they are both rigid transformation.

When an angle is copied to a new angle, the new angle and the old angle will have the same measure

Similarly;

When a line segment is copied to a new segment, the new line segment and the old line segment will have the same measure

This type of transformation is referred to as a rigid transformation.

Hence, copying a line segment is similar to copying an angle, because they are both rigid transformations

Step-by-step explanation:

please help me with this problem

Answers

Answer:

Answer is 544

Hope it helps you

Pete drive 50 miles in 4 hours work out his average speed in miles per hour

Answers

Answer:

Pete's average speed is 12.5 miles per hour

Step-by-step explanation:

Miles per hour means how many miles were driven in 1 hour so :

50 miles in 4 hours = ? miles in 1 hour
4 ÷ x = 1

4/x = 1

x = 4

50÷4 = ?

50/4 = ?

? = 12.5

Pete's average speed is 12.5 miles per hour

Hope this helped and have a good day

Can someone help me with question 3? it geometry, gotta need an answer fast

Answers

Answer:

Step-by-step explanation:

So for the 1st one you're going to use your protractor and line it up so that BC is at 0 degrees, that way you can look at the point A to determine the angle. Now that you have the angle you're going to take your protractor and draw one point which will be where the two lines meet, line this up in the middle and then draw a point at 0 degrees and a point at the angle you found. Now draw another point at half the angle (like in the previous answer I answered). Draw a line from the middle point and connect it to all these three points (4 in total but one of them is going connected to all of them, like the point B in the image you sent).

For the second question you're going to take the angle you found in the first answer and multiply it by 2. Now start off by drawing a middle point which will connect the two other lines. Line up your protractor so that point is at bottom but in the middle (Check my photo for clarification). Now go to the value that is twice the angle and draw a point there, and also draw a point at 0 degrees) and then connect the two points to the middle point.

For question 3 you don't really need the hint, all you need is to multiply the angle by 1.5 but I'll send another image explaining what it's trying to explain.


[tex]x + 6 = 10 \\ x = 10 - 6[/tex]
simultaneous equations

Answers

Answer:

False.

These are not simultaneous equations.

Step-by-step explanation:

Simultaneous equations are equations involving two or more unknowns that are to have the same values in each equation. In the equations given there is only one unknown value.

Explain why statistical acceptance sampling is not an effective method for monitoring process quality.

Answers

Acceptance sampling is a statistical method used to monitor the quality of purchased parts and components. To ensure the quality of incoming parts, a purchaser or manufacturer normally samples 20 parts and allows one defect.

a) Concerned with inspection of products

b) Concerned with decision making regarding products

c) One of the oldest aspects of quality assurance

The Acceptance sampling procedure is necessarily a lot sentencing procedure. It cannot be used to estimate the lot quality or lot conformity to the standard specifications.

The acceptance sampling is used when the test is destructive, or the cost of 100% inspection is quite high, or when we need a continuously monitoring program

The Acceptance sampling procedure is used for decision making of either acceptation or rejection of a lot. It can’t be used as a lot quality estimator

As Acceptance sampling is just a lot sentencing process, it can’t estimate the quality of products in the lot. But designed experiments ensure good quality of the process output before even production

To know more about Statistical acceptance visit:

https://brainly.com/question/13334231

#SPJ4

answer with steps please

Answers

Answer:

Step-by-step explanation:

Answer:  [tex]\frac{-70a^3-195^2-20a-78}{60}[/tex]  or  [tex]-\frac{7}{6} a^3-\frac{13}{4} a^2-\frac{1}{3} a-\frac{13}{10}[/tex]  

When we say subtract this from that, the value or word after from is the one which is ahead of the equation. For example, subtract 9 from 10, then the expression will be 10-9 .  [Answer is 1]

The expression that can be formed from the picture is and the solution is shown in the steps below.

[tex](\frac{a^3}{3} -\frac{3a^2}{4} -\frac{5}{2}) - (\frac{5a^2}{2} +\frac{3a^3}{2} +\frac{a}{3} -\frac{6}{5} )[/tex]           [Formed the expression]
[tex]\frac{a^3}{3} -\frac{3a^2}{4} -\frac{5}{2} - \frac{5a^2}{2} -\frac{3a^3}{2} -\frac{a}{3} +\frac{6}{5}[/tex]                 [Removed the brackets]
[tex]\frac{a^3-a}{3} +\frac{6}{5} -\frac{3a^2}{4} +\frac{-5a^2-3a^3-5}{2}[/tex]       [Brought common denominators together]
[tex]\frac{20(a^3-a)}{60} +\frac{6*12}{60} -\frac{15(3a^2)}{60} +\frac{30(-5a^2-3a^3-5)}{60}[/tex]       [LCM of 3,5,4 and 2]
[tex]\frac{20a^3-20a+72-45a^2-150a^2-90a^3-150}{60}[/tex]                  [Multiplied the numerators]
[tex]\frac{-70a^3-195^2-20a-78}{60}[/tex]                                                    [simplified]
[tex]-\frac{7}{6} a^3-\frac{13}{4} a^2-\frac{1}{3} a-\frac{13}{10}[/tex]                                        [More simplified]

I tried my best to show my steps and solve it. Thank you.

 ∩_∩
(„• ֊ •„)♡
┏━∪∪━━━━┓
    hope it helped
┗━━━━━━━┛

write the equation for the graph given below

Answers

Answer: Option (1)

Step-by-step explanation:

The slope of the line is

[tex]\frac{9-6}{4-(-4)}=\frac{3}{8}[/tex]

This eliminates all the options except for (1).

An approximate solution to an equation is found using this iterative process.
(x₂)³-1
and x₁ = -1
4
Xn+1 =
4/34
a) (i) Work out the value of x₂
(ii) Work out the value of x3
b) Work out the solution to 6 decimal places.

Answers

Answer:

a) i) -0.5

   ii) -0.28125

b)  -0.254102 (6 d.p.)

Step-by-step explanation:

Given iteration formula:

[tex]x_{n+1}=\dfrac{\left(x_n\right)^3-1}{4} \quad \textsf{and} \quad x_1=-1[/tex]

Part (a)(i)

Substitute the value of x₁ into the formula and solve for x₂ :

[tex]\begin{aligned}\implies x_2 & =\dfrac{\left(x_1\right)^3-1}{4}\\\\& =\dfrac{\left(-1\right)^3-1}{4}\\\\ & = \dfrac{-1-1}{4}\\\\ & = \dfrac{-2}{4}\\\\ & = -0.5\end{aligned}[/tex]

Part (a)(ii)

Substitute the value of x₂ into the formula and solve for x₃ :

[tex]\begin{aligned}\implies x_3 & =\dfrac{\left(x_2\right)^3-1}{4}\\\\& =\dfrac{\left(-0.5\right)^3-1}{4}\\\\ & = \dfrac{-0.125-1}{4}\\\\ & = \dfrac{-1.125}{4}\\\\ & = -0.28125\end{aligned}[/tex]

Part (b)

To find the solution to 6 decimal places, keep substituting each new value into the iteration formula until the answers are the same when rounded to the required level of accuracy.

[tex]\implies x_4=-0.2555618286...[/tex]

[tex]\implies x_5=-0.2541728038...[/tex]

[tex]\implies x_6=-0.2541051331...[/tex]

[tex]\implies x_7=-0.2541018552...=-0.254102\:\: \sf (6 \:d.p.)[/tex]

[tex]\implies x_8=-0.2541016964...=-0.254102\:\: \sf (6 \:d.p.)[/tex]

[tex]\implies x_9=-0.2541016888...=-0.254102\:\: \sf (6 \:d.p.)[/tex]

Therefore, the solution is -0.254102 (6 d.p.).

A gardener is planning to fill her garden with mulch. she plots it on a grid to plan how much she will need. the garden is in the shape of a rectangle with vertices at (3, 9) (5, 9) (3, 3) (5, 3). one bag of mulch covers 5ft^2 and costs $5.00. how much will it cost her to cover her garden?

Answers

The cost to cover the garden with mulch will be $12.00, as the area of the rectangular garden is 12 ft.².

The cost of one bag of mulch is given as $5.00.

The area covered by one bag of mulch is 5 ft.².

Therefore, the cost to cover 1 ft.² of the garden with mulch is $(5/5) = $1.00.

To find the area of the rectangular plot representing the garden, we do the follows:

The distance formula between two points (x₁, y₁) and (x₂, y₂) is,

d = √((x₂ - x₁)² + (y₂ - y₁)²).

The length of the rectangular plot between points (3, 9) and (5, 9):

Length = √((5 - 3)² + (9 - 9)²) = √(2² + 0²) = √(4) = 2.

The width of the rectangular plot between the points (3, 3) and (3, 9):

Width = √((3 - 3)² + (9 - 3)²) = √(0² + 6²) = √(36) = 6.

Therefore, the area of the rectangular plot = Length*Width = 2*6 = 12 ft.².

Therefore, the cost to cover the rectangular plot with mulch, at the cost of $1.00 per 1 ft.² = $1*12 = $12.00.

Therefore, the cost to cover the garden with mulch will be $12.00, as the area of the rectangular garden is 12 ft.².

Learn more about area of rectangles at

https://brainly.com/question/12628587

#SPJ4

8 times the number is 200. What is 80 times the number?

Answers

Answer:

2,000

Step-by-step explanation:

⇒ 200/8 = 25

⇒ 25 * 80 = 2,000

Answer:

2000.

Step-by-step explanation:

8 * x = 200

x = 200/8

80x = 80 * 200/8

       = 10 *200

       = 2000.

 

The table and Circle Chart display proportions of the expenditure of a local government. Which statement describes how the Circle Chart misrepresents the data?


A. The sum of the percentages in the sections is not 100.
B. The size of each section does not correspond to the percent expenditure for each category given in the table.
C. One of the categories in the table is not represented in the circle chart.
D. The size of the section representing highways is greater than the size of the section representing education.

Answers

This exercise refers to the identification of statistics that is misleading or erroneous. In this case, a circle graph and a table are involved.

What is a circle graph?

A circle graph is simply a pie chart. It is used to depict information in percentages. An example is attached.

Although charts and tables are very useful descriptive and or illustrative statistics, they can be misleading.

This can happen when:

The sum of the percentages in the sections is not 100;The size of each section does not correspond to the percent expenditure for each category given in the table; andOne of the categories in the table is not represented in the circle chart.

This answer is general in nature due to the limited information provided.

Learn more about circle graphs at;
https://brainly.com/question/308526
#SPJ1

Answer:

C

Step-by-step explanation:

1. How could you figure out the coordinates of a reflection over the x-axis? What would the coordinates of that reflected point be?

2. Then also tell how you could determine the coordinates when (-8,2) is reflected over the y-axis, and tell the coordinates of that new point.

Answers

The reflection of a point over the x axis is given by the equation (x, y) ⇒ (x, -y).

If the point (-8,2) is reflected over the y-axis, the new point would be (8, 2)

What is a transformation?

Transformation is the movement of a point from its initial location to a new location. Types of transformation are reflection, translation, rotation and dilation.

The reflection of a point over the x axis is given by the equation (x, y) ⇒ (x, -y).

If the point (-8,2) is reflected over the y-axis, the new point would be (8, 2)

Find out more on transformation at: https://brainly.com/question/4289712

#SPJ1

Other Questions
After reading the excerpts from The Struggle for Human Rights by Eleanor Roosevelt, annotate the text using the strategies that you learned. Fill in the table below with your annotations. need help pls There is a large park which is owned by the local city government with a food forest containing many fruit and nut trees. What will most likely happen to that food forest Fertility in humans is a trait that is controlled by multiple genes, and which has environmental influences, such as the health and nutrition of the potential parents. What terms describe fertility as a trait How many colvant bonds can a typical carbon atom form la mama de fabiana tiene un vivero. En su ultimo pedido compro 5 fresias, una con flores rojas, una con flores blancas, una con flores celestes y otra con flores violetas, 3 rosales, una con flores rojas, otro con flores rosas y otro con flores amarillas y 2 calas, una con flores blancas y otra con flores azules.Me ayudan porfa.Les doy puntos y coronita si me ayudan.Porfa. . A bus covers 44 kilometres in 3 2/3 -hours. How much distance it has covered in 1 hour ? please need fast Today only, a table is being sold for $100.80. This is 24% of its regular price. What was the price yesterday? Scobie Company began 2016 with a retained earnings balance of $142,400. During an examination of its accounting records on December 31, 2016, Scobie found it had made the following material errors, for both financial reporting and income tax reporting, during 2015.1. Depreciation expense of $15,000 inadvertently had been recorded twice for the same machine.2. No accrual had been made at year-end for interest; therefore, interest expense had been understated by $4,000.Scobies net income after taxes during 2016 was $60,000. The company has been subject to a 30% income tax rate for the past several years. It declared and paid dividends of $13,000 during 2016.Required:1. Prepare whatever journal entries in 2016 are necessary to correct Scobies books for its previous errors. Make your corrections directly to the Retained Earnings account. 1. Name a pair of supplementary angles in this figure. angles FEH and KED angles FEK and FEH angles FEK and FEJ angles FEJ and JED During the ________ stage of team development, team members shift their attention away from task orientation to a relationship focus. Group of answer choices storming forming norming adjourning Edit the text.Add the correct end of sentencepunctuation marks to each of thesesentences."Shark!" shouted the lifeguard,his voice cracking as he did soUnder a so-called "latte levy",people could be charged 25p forpaper coffee cupsI wonder what she will say whenshe gets hereHave you been waiting long Fitness Your goal is to take at least 10,000 steps per day. According to your pedometer, you have walked 527 4 steps. Write and solve an inequality to find the possible numbers of steps you can take to reach your goal. Suppose the mass of a fully loaded module in which astronauts take off from the Moon is 10,600 kg. The thrust of its engines is 25,500 N. (Assume that the gravitational acceleration on the Moon is 1.67 m/s2.)(a) Calculate its magnitude of acceleration in a vertical takeoff from the Moon. m/s2(b) Could it lift off from Earth? If not, why not?Yes, the thrust of the module's engines is equal to its weight on Earth.No, the thrust of the module's engines is equal to its weight on Earth. No, the thrust of the module's engines is less than its weight on Earth.Yes, the thrust of the module's engines is greater than its weight on Earth.If it could, calculate the magnitude of its acceleration. (If not, enter NONE.) m/s2 Which body region should be avoided during myofascial release techniques? Lumbar spine Thoracic spine Knee Pelvis Given the different combinations of goods and services a consumer can obtain within his or her budget, the ______ can determine the specific combination that will yield the most utility or satisfaction. how is this true?......... How can you expand your resume if you only limited formal work experience seminoma of the testicle generally appears on ultrasound as a(n) _____________ mass. * 5 points Hypoechoic Hyperechoic Anechoic Complex What is the slope of a line perpendicular to 3x + 2y = 7?-2/3-3/23/22/3 The effects that a change in market conditions, usually price, has on the welfare or economic well-being of market participants are